Difficult integral involving exponential

Click For Summary
The discussion centers on verifying the Fourier transform of the function f(x) = e^{-ax^2}, with the proposed transform being \hat{f}(k) = \frac{1}{\sqrt{2a}}e^{-k^2/4a}. The user encounters difficulties with the integration process, particularly in completing the square for the exponent in the integral. There is a suggestion that the provided transform may be incorrect, as the correct form should be \hat{f}(k) = \sqrt{\frac{\pi}{a}}e^{-k^2/4a}. Additionally, a minor point of contention arises regarding the sign in the exponential of the Fourier transform integral.
zandria
Messages
15
Reaction score
0

Homework Statement


I'm trying to verify the Fourier transform but am getting stuck on the integration. Here is the pair:
f(x) = e^{-ax^2}
\hat{f}(k) = \frac{1}{\sqrt{2a}}e^{-k^2/4a}
a>0

Homework Equations



I know that
\hat{f}(k)=\int_{-\infty}^{\infty}f(x)e^{ikx}dx

The Attempt at a Solution



So I have
\hat{f}(k)=\int_{-\infty}^{\infty}e^{-ax^2}e^{ikx}dx
\hat{f}(k)=\int_{-\infty}^{\infty}e^{-ax^2+ikx}dx

I tried using integration by parts and I'm not sure that's the right way to go. If it is I'm not sure how to go about it without getting a more complicated integral.
 
Physics news on Phys.org
You need to complete the square, which means:

-(ax^2+ikx)=-\left[(\alpha x+\beta)^2+\gamma \right].

Find \alpha,\beta and \gamma.

Edit1: it seems you have either listed \hat{f}(k) wrong or the book where you got \hat{f}(k) from is wrong, because the answer should be:

<br /> \hat{f}(k) = \sqrt{\frac{\pi}{a}}e^{-k^2/4a}<br />

edit2: While making no difference to the final answer in this case, shouldn't it be \hat{f}(k)=\int_{-\infty}^{\infty}f(x)e^{-ikx}dx, note the minus sign.
 
Last edited:
Question: A clock's minute hand has length 4 and its hour hand has length 3. What is the distance between the tips at the moment when it is increasing most rapidly?(Putnam Exam Question) Answer: Making assumption that both the hands moves at constant angular velocities, the answer is ## \sqrt{7} .## But don't you think this assumption is somewhat doubtful and wrong?

Similar threads

Replies
5
Views
2K
  • · Replies 5 ·
Replies
5
Views
2K
  • · Replies 3 ·
Replies
3
Views
2K
  • · Replies 2 ·
Replies
2
Views
2K
Replies
1
Views
2K
Replies
8
Views
2K
  • · Replies 7 ·
Replies
7
Views
2K
Replies
6
Views
2K
  • · Replies 14 ·
Replies
14
Views
2K
  • · Replies 3 ·
Replies
3
Views
2K